What is the range for this set of data?
38 17.55 40
O 2
O38
O39
O72

Answers

Answer 1

Answer:

The original price of the shirt is simply the sum of the purchased price and the discount price, that is:

original price = $17.55 + $5.85

original price = $23.40

The equation that best model this scenario is:

x - 5.85 = 17.55

where x is the original price

Step-by-step explanation:


Related Questions

Find the value of x 90, 68 2x

Answers

Answer:

2x + x +90= 180 We will add 2x + x=3x We get 3x + 90 =180 Now we subtract 3x = 180–90 ... x=30, for confirmation put value of x =30in equation and verify. LHS=RHS. Thats it. 68 views.


Write the number 16408 in words.

Answers

Answer:

Step-by-step explanation:

Sixteen thousand, four hundred and eight

Hope this helps!

write the equation for the function of the graph below​

Answers

Answer:  (1) [tex]y= \sqrt{x+2}-2[/tex]   (2) y = (x - 3)³

                    D: x ≥ -2  [-2, ∞)      D: -∞ < x < ∞ (-∞, ∞)

                    R: y ≥ -2  [-2, ∞)      R: -∞ < y < ∞ (-∞, ∞)

                    Inc: x > 2 (2, ∞)      Inc: (-∞, 3) ∪ (3, ∞)

                    Dec: never             Dec: never

Step-by-step explanation:

NOTES:

Domain is the x-values.

Range is the y-values.

From left to right: increasing when going up and decreasing when going down. The anchor and turning point are not considered to be increasing or decreasing.

Jenny sold 17 less than 3 times the number of quilts Laura sold. Jenny sold
4 quilts. Select whether each statement is true or false. ZEE.4. ZEE 4a

Answers

Answer:

the statement would be false.

Step-by-step explanation:

Jenny sold 17 less than 3 times lauras 4 quilts. So that would make Laura have 12 and 12<17.

Help plz:)))I’ll mark u Brainliest

Answers

Answer:

18 foot ladder will reach at 14.4 feet up the wall.

Step-by-step explanation:

Triangles formed by the ladder against wall and the ground are similar,

ΔABC ~ ΔDEC

Therefore, their corresponding sides will be proportional.

[tex]\frac{AB}{DE}= \frac{BC}{EC}= \frac{AC}{DC}[/tex]

[tex]\frac{AB}{DE}= \frac{BC}{EC}[/tex]

[tex]\frac{18}{10}=\frac{BC}{8}[/tex]

BC = [tex]\frac{18\times 8}{10}[/tex]

     = 14.4

Therefore, 18 foot ladder will reach at 14.4 feet up the wall.

J.B. earns $16.60 an hour with time-and-a-half for hours worked over 8 a day. His

hours for a week are 8.25, 8.25, 9.5, 11.5, and 7.25. Determine his gross earnings

for that week.

Answers

Answer:

The gross pay for the week is $701.32

Step-by-step explanation:

J.B. earns $16.60 per hour, and half of it ($16.60/2) for each hour worked over the 8th-hour range.

On the first day, he works 8.25 hours.

We have 8 hours, plus 0.25 hours past the limit, then for the first day he wins:

8*$16.60 + 0.25*($16.60/2) = $134.86

On the second day, he works again 8.25 hours, so he wins the same amount $134.86

On the third day, he works 9.5 hours, then:

We have 8 hours plus 1.5 hours past the limit, then for this day he wins:

8*$16.60 + 1.5*$16.60/2 = $145.25

On the fourth day, he works 11.5 hours, then:

We have 8 hours plus 3.5 hours pást the limit, then for this day he wins:

8*$16.60 + 3.5*$16.60/2 = $161.85

For the last day, he works 7.25 hours, (in this case, we do not have hours worked over the 8-hour limit) then this day he wins:

7.25*$16.60 = $124.50

The gross pay for this week will be equal to:

G = $134.86 + $134.86 +  $145.25 + $161.85 + $124.50 = $701.32

work out the perimeter of the semi-circle take pi to be 3.142​

Answers

Answer:
P = 25.71 cm
Step-by-step explanation:
Perimeter of semicircle = πD/2
Where D=10 cm
P = (3.142)(10)/2
P = (3.142)(5)
P = 15.71 cm
Complete Perimeter of Semicircle = P + Diameter
= 15.71 + 10
= 25.71 cm

Given f(x)=x^3-2, find the equation of the secant line passing through (-4,f(-4)) and (2,f(2))

Answers

Given:

The function is

[tex]f(x)=x^3-2[/tex]

The secant line passing through (-4,f(-4)) and (2,f(2)).

To find:

The equation of the secant line.

Solution:

We have,

[tex]f(x)=x^3-2[/tex]

At x=-4,

[tex]f(-4)=(-4)^3-2[/tex]

[tex]f(-4)=-64-2[/tex]

[tex]f(-4)=-66[/tex]

At x=2,

[tex]f(2)=(2)^3-2[/tex]

[tex]f(2)=8-2[/tex]

[tex]f(2)=6[/tex]

The secant line passes through the points (-4,-66) and (2,6). So, the equation of the secant line is

[tex]y-y_1=\dfrac{y_2-y_1}{x_2-x_1}(x-x_1)[/tex]

[tex]y-(-66)=\dfrac{6-(-66)}{2-(-4)}(x-(-4))[/tex]

[tex]y+66=\dfrac{6+66}{2+4}(x+4)[/tex]

[tex]y+66=\dfrac{72}{6}(x+4)[/tex]

On further simplification, we get

[tex]y+66=12(x+4)[/tex]

[tex]y+66=12x+48[/tex]

[tex]y=12x+48-66[/tex]

[tex]y=12x-18[/tex]

Therefor, the equation of the secant line is [tex]y=12x-18[/tex].

Exercise 2.4.2: Proving statements about rational numbers with direct proofs. About Prove each of the following statements using a direct proof. (a) The product of two rational numbers is a rational number. Solution (b) The quotient of a rational number and a non-zero rational number is a rational number. Solution (c) If x and y are rational numbers then is also a rational number.

Answers

Answer:

See Explanation

Step-by-step explanation:

(a) Proof: Product of two rational numbers

Using direct proofs.

Let the two rational numbers be A and B.

Such that:

[tex]A = \frac{1}{2}[/tex]

[tex]B = \frac{2}{3}[/tex]

The product:

[tex]A * B = \frac{1}{2} * \frac{2}{3}[/tex]

[tex]A * B = \frac{1}{1} * \frac{1}{3}[/tex]

[tex]A * B = 1 * \frac{1}{3}[/tex]

[tex]A * B = \frac{1}{3}[/tex]

Proved, because 1/3 is rational

(b) Proof: Quotient of a rational number and a non-zero rational number

Using direct proofs.

Let the two rational numbers be A and B.

Such that:

[tex]A = \frac{1}{2}[/tex]

[tex]B = \frac{2}{3}[/tex]

The quotient:

[tex]A / B = \frac{1}{2} / \frac{2}{3}[/tex]

Express as product

[tex]A / B = \frac{1}{2} / \frac{3}{2}[/tex]

[tex]A / B = \frac{1*3}{2*2}[/tex]

[tex]A / B = \frac{3}{4}[/tex]

Proved, because 3/4 is rational

(c) x + y is rational (missing from the question)

Using direct proofs.

Let x and y be

Such that:

[tex]x = \frac{1}{2}[/tex]

[tex]y = \frac{2}{3}[/tex]

The sum:

[tex]x + y = \frac{1}{2} + \frac{2}{3}[/tex]

Take LCM

[tex]x + y = \frac{3+4}{6}[/tex]

[tex]x + y = \frac{7}{6}[/tex]

Proved, because 7/6 is rational

The above proof works for all values of A, B, x and y; as long as they are rational values

guys help. show work please

Answers

9514 1404 393

Answer:

  2^17×3^7

Step-by-step explanation:

The applicable rules of exponents are ...

  (a^b)^c = a^(bc)

  (a^b)(a^c) = a^(b+c)

__

  [tex](2^9\times3^5)\times(2^4\times3)^2=2^9\times3^5\times2^{4\cdot2}\times3^2\\\\=2^{9+8}\times3^{5+2}=\boxed{2^{17}\times3^7}[/tex]

Find a cubic polynomial that goes through points (4, – 22) and (3, - 26) and has tangents with slopes
respectively 11 and — 2 there. Check your work with a graphing utility.
f() =

Answers

Let f(x) = ax ³ + bx ² + cx + d.

The graph of f(x) passes through (4, -22) and (3, -26), which means f (4) = -22 and f (3) = -26, so that

64a + 16b + 4c + d = -22

27a + 9b + 3c + d = -26

When the question says it has tangents at some point, I take that to mean the slope of the tangent line at that point is the given number. So f ' (4) = 11 and f ' (3) = -2. We have

f '(x) = 3ax ²+ 2bx + c

so that

48a + 8b + c = 11

27a + 6b + c = -2

Solve the system:

• Eliminate d :

(64a + 16b + 4c + d) - (27a + 9b + 3c + d) = -22 - (-26)

→   37a + 7b + c = 4

• Eliminate c :

(48a + 8b + c) - (27a + 6b + c) = 11 - (-2)

→   21a + 2b = 13

(48a + 8b + c) - (37a + 7b + c) = 11 - 4

→   11a + b = 7

• Eliminate b, then solve for a and the other variables:

(21a + 2b) - 2 (11a + b) = 13 - 2 (7)

-a = -1

a = 1   →   b = -4   →   c = -5   →   d = -2

Then

f(x) = x ³ - 4x ² - 5x - 2


Answer:
Need help with this plz

Answers

Answer:

(a) 3x+28+5x+52+2x-10=180 (∠ sum of Δ)

                          10x+70=180

                                 10x=110

                                     x=11

(b) ∠C = 2x-10

           = 2(11)-10

           =  12°

In the slope 5/9, the 5 indicates to

A. Rise 5 units
B. Run 5 units
C. Fall 5 units

Answers

A Bc it rise 5 units

Triangle R Q S is cut by line segment T V. Line segment T V goes from side Q R to side R S. The length of R V is x + 10, the length of V S is x, the length of R T is x + 4, and the length of T Q is x minus 3. Which value of x would make Line segment T V is parallel to Line segment Q S?
PLEASE HURRY

Answers

9514 1404 393

Answer:

  x = 10

Step-by-step explanation:

In order for TV to be parallel to QS, it must divide the sides of the triangle proportionally.

  RT/TQ = RV/VS

  (x+4)/(x-3) = (x+10)/(x)

  1 +7/(x-3) = 1 +10/x . . . . expand each fraction

  7/(x -3) = 10/x . . . . . . . . subtract 1

  7x = 10(x -3) . . . . . . . . . cross multiply

  30 = 3x . . . . . . . . . . . . add 30-7x, simplify

  10 = x . . . . . . . . . . . . . . divide by 3

The value of x that makes the segments parallel is 10.

_____

Alternate solution

You can cross multiply the first fraction we wrote to get ...

  x(x +4) = (x -3)(x +10)

  x^2 +4x = x^2 +7x -30 . . . eliminate parentheses

  30 = 3x . . . . . . subtract x^2+4x-30 from both sides

Answer: C. 10

Step-by-step explanation:

i got it right on the test

-1/2x-(1/2x+4)+12=17x-6(3x+5/6) What value of x makes the equation true? Show your work please.

Answers

Answer:

No Solution

Step-by-step explanation:

Keep in mind that if the grouping isn't right, then my result will be wrong:

-1/2x-(1/2x+4)+12=17x-6(3x+5/6)

Let me just change the formatting for convenience, and then begin the calculations:

[tex]-\frac{1}{2}x-\left(\frac{1}{2}x+4\right)+12=17x-6\left(3x+\frac{5}{6}\right)\:[/tex]

[tex]-\frac{1}{2}x-\frac{1}{2}x-4+12=17x-18x-5[/tex]

[tex]-2\cdot \frac{1}{2}x-4+12=17x-18x-5[/tex]

[tex]-x-4+12=17x-18x-5[/tex]

[tex]-x+8=17x-18x-5[/tex]

[tex]-x+8=-x-5[/tex]

[tex]-x=-x-13[/tex]

[tex]0=-13[/tex]

Result: No Solution

Amber has determined that the experimental probability of making a free throw in basketball is 12/15. What it the probability of missing a basket?

Answers

Answer:

3/15, 1/5, or 0.2 probability. Highly unlikely

Explanation:

12/15 is the shown probability. The leftover is 3/15, so 3/15 is the chance of missing the basket.

~Hope this helps~

Alison rolls two number cubes 90 times. How many times is he going to get a sum of 3?

Answers

15

Step-by-step explanation:

Answer:

I think 30 im so sure

Step-by-step explanation:

uhm i think its 90 divided by 3 cuz i looked it up and got 30 so yea bye

The area of a circle is 100π ft². What is the circumference, in feet? Express your answer in terms of π.

Answers

Answer:

Answer: 20pi is the circumference

Step-by-step explanation:

The circumference of the circle is 20π ft when the area of the circle is 100π ft².

What is circle?

A circle is a locus of the points drawn at an equidistant from a fixed point. The fixed point is called Centre of the circle and the distance from the fixed point to the circle is called radius.

Given, area of a circle is 100π.

Therefore,  πr² =100π, where r is the radius of the circle.

So, r²=100

Thus, r= 10 ft.

now, circumference = 2πr

                                  =2π×10

                                  =20π ft.

To learn more about circle click here:

https://brainly.com/question/21280267

#SPJ3

Zack has an old car. He wants to sell it for 60% off the current price. The market price is $500. How much money would he receive in exchange for the car if he were able to sell it at that rate? What if he wanted to sell it for 25% more than the market price?

Answers

300 and 625

60% of 500 is 300

So if Zack wanted to sell the car 60% of the markets price, it would be 300.

125% of 500 is 625

If Zack wanted to sell the car for 25 more than the price at market, it would be 125%, and 125% of 500 is 625.

Jamie has white, blue, and pink shirts. She has black, white, and beige pants. She has black and brown shoes. If she chooses one shirt,

one pair of pants, and one pair of shoes, how many unique outfits can she make?

Answers

Answer:

She can make [tex]18[/tex] unique outfits.

Step-by-step explanation:

Jamie has [tex]3[/tex] options to choose from for a shirt (white, blue, or pink), [tex]3[/tex] options to choose from for pants (black, white, or beige), and [tex]2[/tex] options to choose from for shoes (black or brown). Therefore, she can make [tex]3*3*2=18[/tex] unique outfits from these options. Hope this helps!

Find the slope of the line passing through the points (-2,-3) and (2,5) 

Answers

Answer:

Undefined

Step-by-step explanation:

Find the measure of CD.
mCD= [?] degrees

Answers

Given:

A circle with radius 7 units and chord CD = 7 units.

To find:

The measure of arc CD.

Solution:

Let O be the center of the circle.

In triangle OCD,

[tex]OC=7[/tex]            (Given)

[tex]OC=OD=7[/tex]          (Radii of same circle)

[tex]CD=7[/tex]            (Given)

Since [tex]OC=CD=OD[/tex] all sides are equal, therefore, the triangle OCD is an equilateral triangle.

Measure of each angle of an equilateral triangle is 60 degrees. So,

[tex]m\angle COD=60^\circ[/tex]

The measure of central angle is equal to the measure of corresponding arc.

[tex]m(arcCD)=m\angle COD[/tex]

[tex]m(arcCD)=60^\circ[/tex]

Therefore, the measure of arc CD is 60 degrees.

Use the diagram below to answer the following questions.
The infield of a baseball field is a square. The distance from home plate to first base is 90 ft.
2nd
8a. What is the distance from home plate to second base?
pitchers mound
3rd
1st
90 ft
8b. What is the distance from third base to first base?
Home
8c. If the pitcher's mound is 60 ft 6 in from home plate, is it the midpoint of the diagonal from home
plate to second base?

Answers

Answer:

Step-by-step explanation:

Distance from home plate to second base = 90√2 feet

Distance from third base to first base = 90√2 feet

60 ft 6 in = 60.5 ft

90√2 /2 = 45√2 ≅ 63.6 ft ≠ 60.5 ft

The pitcher's mound is not at the midpoint.

A recent drug survey showed an increase in use of drugs and alcohol among local high school students as compared to the national percent. Suppose that a survey of 100 local youths and 100 national youths is conducted to see if the percentage of drug and alcohol use is higher locally than nationally. Locally, 65 seniors reported using drugs or alcohol within the past month, while 61 national seniors reported using them. Conduct a hypothesis test at the 5% level.

Answers

Answer:

The answer is "0.586"

Step-by-step explanation:

Please find the solution to the given question:  

[tex]z= \frac{0.65-0.61}{\sqrt{\frac{0.65 \times 0.35}{100}+\frac{0.61 \times 0.39}{100}}}[/tex]

  [tex]= \frac{0.04}{\sqrt{\frac{0.2275}{100}+\frac{0.2379}{100}}}\\\\= \frac{0.04}{\sqrt{0.002275+0.002379}}\\\\= \frac{0.04}{\sqrt{0.004654}}\\\\= \frac{0.04}{0.0682202316}\\\\=0.586336327\\\\[/tex]

PLZ HELP ME YOU GUYS ARE MY ONLY HOPE!!! I WILL GIVE BRAINLIEST PLZ ANSWER<3
2a+3c=5a+2c=1

There is an ordered pair that is the solution of the system of equations shown. The value of one of the variables in the solution of the system can be obtained by using which of the following substitutions?

Answers

Answer:

a = 9, c = -4?

Step-by-step explanation:

i think these are the two equations, let me know if i'm wrong

2a + 3c = 5; a + 2c = 1

rearrange second equation : a = -2c +1

plug in:

2(-2c +1) +3c = 5

-4c +1 +3c = 5

-c +1 = 5

-c = 4

c = -4

a + 2(-4) = 1

a - 8 = 1

a = 9

a=9, c=4 is the answer good luck

The histogram on the left shows the number of hours students in a British Literature class read last week, rounded to the nearest hour. Which of the following statements offers the best description of the median number of hours students in the class read last week?​

Answers

Answer:The median number of hours is between 5 and 9

Step-by-step explanation:

The median number of hours students in the class read last week is between 5 and 9

Histogram A histogram is an approximate representation of the distribution of numerical dataHistogram is a diagram consisting of rectangles whose area is proportional to the frequency of a variable and whose width is equal to the class interval.

How to solve this problem?

The steps are as follow:

Give class interval; Hours of reading

0-4;20

5-9;24  32th median

10-14;10

15-19;4

20-24;2

25-29;3    ∑ = 63 i.e. odd

M = 1/2 (63+1) = 32th observation

So the median number of hours students in the class read last week is between 5 and 9

Learn more about Histogram here:

https://brainly.com/question/2962546

#SPJ2

WILL GIVE BRAINLIEST

Answers

Answer:

[tex]\frac{y}{8}[/tex]

Step-by-step explanation:

quotient means division

the / means divide

Answer:

y/8

Step-by-step explanation:

quotient means divide, so y divided by 8, or y/8

A company has a linear total cost function and has determined that over the next three
months it can produce 13,000 units at a total cost of $226,000. This same manufacturer
can produce 16,000 units at a total cost of $296,000. The sales price per unit is $31.25.
i. Find the revenue, cost, and profit functions using q for number of units.
ii. Find the marginal cost, average cost and fixed cost.
iii. Find break-even quantity.

Answers

I am so sorry but I don’t really know the answrr

I don’t know what to do please help

Answers

Answer:

f(2)=0

Step-by-step explanation:

You plug in 2 for any X's in the equation

f(2)=2^2-4

If f(x)=x^2-4 and x=2
Then f(2)=2^2-4.
= 4-4
=0
Don’t write this in answer just to explain:the value of x is 2 so in the eq
X^2-4 so we write 2 where the x is
= 2^2-4 ...now solve it as above


Donna guessed there were 400 jellybeans in a jar. There were really 480 jellybeans. What is her prevent of error

Answers

Answer:

Step-by-step explanation:

Donna guessed 400 but there were 480 so she was off by 80

Other Questions
The diagram below provides details about Florida in the late 19th century. Steps in Computing The Median Absolute Deviation Choose the sentence that is grammatically correct.Nosotros deber compramos libros nuevos.Nosotros debemos comprar libros nuevos.Nosotros debemos compramos libros nuevos.Nosotros deber comprar libros nuevos. Does the point (7, 4) satisfy the equation y = x 3? Benjamin Franklin wasn't just a political leader. What else was he known for? What transformations are applied to the graph of the function f(x) = 10^x to produce the graph of the function g(x) = 3(10)^x 2?A. a vertical dilation by a factor of 1/3 and a vertical shift down 2 unitsB. a vertical dilation by a factor of 3 and a horizontal shift to the right 2 unitsC. a vertical dilation by a factor of 3 and a vertical shift down 2 unitsa vertical dilation by a factor of and a horizontal shift to the right 2 unitsOB.OC.OD The first three terms of a sequence are given. Round to the nearest thousandth (if necessary). 19, 38,76,... 19,38,76,... \text{Find the 8th term.} Find the 8th term. write x^3-11x^2 in standard form 1) In 1750, which country posed the most serious threat to the English colonies in North America? 2) Which group of people did Samuel Adams create that helped build outrage among other colonists?3) Why did many colonists protest the Stamp Act?4) What reason did Parliament give for raising taxes in the colonies after 1763? 5) What is one way that Britain punished Massachusetts after the Boston Tea Party?6) What did King Georges Proclamation of 1763 do in North America? 7) What immediate action did the Second Continental Congress take as a result to the fighting at Lexington and Concord?8) What are the three main ideas in the Declaration of Independence? 9) What is the idea of natural rights? 10) According to the Declaration of Independence, the people have the right to alter or abolish a government if that government does what? 11) Which battle became a turning point of the war because it prompted the French to join the Patriots?12) When Richard Henry Lee said he was now convinced of the necessity of separation, he was describing the influence of which document that we studied? 13) Thomas Paine is known for writing which document that we studied? 14) What was a major advantage of the American side in the Revolutionary War?15) The American victory in the Revolutionary War led to what taking place around the world? Short Answer Topic/Questions:1) You will be given a document to read and answer the question Did the American Revolution Accomplish its Goals? HELP ASAPPlzzzzzzz PLEASE HURRY PLEASE I WILL MARK BRAINLIEST!!!!!!! PLEASE ANSWER QUICKLY!!! What does buying stock on margin mean? Buying stock for a fraction of its cost and borrowing against future profits. Buying stocks after receiving a tip that the stock price will go up. Buying stock on behalf of another person. Buying stock on credit, putting on money down initially. The following chemical reaction takes place in aqueous solution: SnSO 4 (aq)+2 NaOH(aq) Sn(OH) 2 (s)+Na 2 SO 4 (aq) Write the net tonic equation for this reaction. 2. Who are the ultimate "deciders" of what's Constitutional? In other words,who has the last word on whether laws follow the Constitution? 9 - 3x +7y - +3Thx!!! what is unequal reproductive success Which of the following rhythms take the pattem eighth, eighth, quarter, half? 30 points. Given 450.98 g of Cu(NO3)2, how many moles of Ag can be made? Provide your final answer rounded to two decimal places.Cu + 2 AgNO3 Cu(NO3)2 + 2 Ag Restate the questionsAnswer in a complete sentenceProvide textual evidence from the passageQuestion 1. Give three examples from the text which support Muhammad Alis statement that, I have always believed in myself.Question 2. How was Muhammad Alis will tested after he retired?Poem:I have always believed in myself, even as a young childgrowing up in Louisville, Kentucky. My parents instilled a sense ofpride and confidence in me, and taught me and my brotherthat we could be the best at anything. I must have believedthem, because I remember being the neighborhood marblechampion and challenging my neighborhood buddies tosee who could jump the tallest hedges or run a foot racethe length of the block. Of course I knew when I made thechallenge that I would win. I never even thought of losing.In high school, I boasted weekly if not daily thatone day I was going to be the heavyweight champion of theworld. As part of my boxing training, I would run downFourth Street in downtown Louisville, darting in and out oflocal shops, taking just enough time to tell them I wastraining for the Olympics and I was going to win a goldmedal. And when I came back home, I was going to turnpro and become the world heavyweight champion inboxing. I never thought of the possibility of failing onlyof the fame and glory I was going to get when I won. Icould see it. I could almost feel it. When I proclaimed that Iwas the "Greatest of All Time" I believed in myself. And I still do.Throughout my entire boxing career, my belief in myabilities triumphed over the skill of an opponent. My willwas stronger than their skills. What I didn't know was thatmy will would be tested even more when I retired.In 1984, I was conclusively diagnosed with Parkinson'sdisease. Since that diagnosis, my symptoms have increasedand my ability to speak in audible tones has diminished. Ifthere was anything that would strike at the core of myconfidence in myself, it would be this insidious disease. Butmy confidence and will to continue to live life as I choosewon't be compromised.Early in 1996, I was asked to light the caldron at theSummer Olympic Games in Atlanta. Of course myimmediate answer was yes. I never even thought of havingParkinson's or what physical challenges that would present for me.When the moment came for me to walk out on the140-foot-high scaffolding and take the torch from JanetEvans, I realized I had the eyes of the world on me. I alsorealized that as I held the Olympic torch high above myhead, my tremors had taken over. Just at that moment, Iheard a rumble in the stadium that became a poundingroar and then turned into a deafening applause. I wasreminded of my 1960 Olympic experience in Rome, when Iwon the gold medal. Those 36 years between Rome andAtlanta flashed before me, and I realized that I had come full circle.Nothing in life has defeated me.I am still the "Greatest." This I believe.